homeomorphic to Rn, and G* is a union of at most l1 disjoint families. Since a subspace homeomorphic to Rn can only meet countably many disjoint open sets, each G E G* meets at most l1 members of G*. Finally, consider the equivalence relation on g* in which G, G' E G* are equivalent if and only if there exists a finite sequence Go, ... , Gk E G* such
that Go = G, Gk = G', and Gi n Gi+l # 0 (i = 0,1, ... , k - 1). Using the result of the last paragraph, induction on k shows that every equivalence class contains at most R1 sets from g*. On the other hand, if G C G* is an equivalence class, then UG is open and closed in X, and then g = g* by connectedness of X. Therefore, the cardinality of g* is at most 1Z1.
Thus, X is a union of at most R1 subsets homeomorphic to Rn, and therefore it is a union of at most l1 compact sets.
3.10 TOPOLOGY
545
Remarks. 1. The most well-known nonmetrizable topological manifold, the long line, is an example of a space that satisfies the hypotheses of the problem but is not a union of count ably many compact subspaces. 2. Several contestants proved the statement of the problem using the assumption that all separable subspaces of X are second-countable. Gabor Moussong showed that there exists a model of ZFC where this assump-
tion holds. Note, however, that by a construction of M. E. Rudin and P. H. Zenor in some other models of ZFC there exist separable and not second-countable topological manifolds that satisfy the hypotheses of the problem. 3. Gabor Moussong noticed that the statement of the problem follows from the continuum hypothesis, since the cardinality of a connected topological manifold of dimension > 1 is continuum.
Problem T.17. A map F : P(X) -> P(X), where P(X) denotes the set of all subsets of X, is called a closure operation on X if for arbitrary A, B C X, the following conditions hold: (i) A C F(A);
(ii) A C B = F(A) C F(B); (iii) F(F(A)) = F(A). The cardinal number min{ CAI : A C X, F(A) = X} is called the density ofF and is denoted by d(F). A set H C X is called discrete with respect
to F if u
F(H - {u}) holds for all u E H. Prove that if the density of the closure operation F is a singular cardinal number, then for any nonnegative integer n, there exists a set of size n that is discrete with respect to F. Show that the statement is not true when the existence of an infinite discrete subset is required, even if F is the closure operation of a topological space satisfying the T1 separation axiom.
Solution. (a) We prove the statement by induction on n. For n = 0, the statement is obvious; assume that it is true for n. Let the density of the closure
operation F on X be A with cf (A) = n < A. Let JAI = A, with F(A) = X, and well-order A in order type A. By recursion, define a sequence B = {x : < Al by x£ = min (A - F ({x,, : q < l;})). Then F(B) D F(A) = X. Let C C B such that ICI = , and C is cofinal in B, and for Y C B put F'(Y) = F(Y U C) fl B. It is easy to see that F' is a closure operation on B and that d(F') _ A. By the induction hypothesis, there exists a set {x£; : i < n} that is discrete with respect to F'. Let Cn > i (i < n) be such that xC E C. Then the set {xC, i < n} is discrete since xC V F ({x£; : i < n}) C F ({xn : q < En}), and x£i V F ({x£, : i # j, j < n}) C :
F'({xC3 :i# j, j
546
3. SOLUTIONS TO THE PROBLEMS
exists by Zorn's lemma.) Call F C A closed if, for each H E N, IF n HI = w implies F D H. It is easy to see that by this a T1 topological space is defined in which there are no infinite discrete subsets. It remains to prove that the density is A. Let A0 C A, with IAoI < A. For < wl, we define a sequence A by the following recursion: if EN:IHnA,.I =w}, and if is a limit =71+1, then ordinal, then put A£ = U{Aq :77 < }. Since IA,7+11 _< IAqIw, it is easy to see that IACI < IAoIw < A for < wl. On the other hand, A,,,, is obviously closed, and therefore A0 is not dense.
Problem T.18. Suppose that K is a compact Hausdorff space and K = U'--OA,,, where An is metrizable and An C A,,,, for n < m. Prove that K is metrizable. Solution. First, we prove the following lemma. Lemma. If a subspace H of a metric space A is not separable, then there exists an uncountable, discrete, closed subspace Z C H.
Proof. Let Zn be a maximal subset of A such that d(x, y) > 1/n for all x, y E Zn, x # y. If a E A, then d(a, x) < 1/2n can hold for at most one point of Zn, so Zn is discrete. On the other hand, by maximality of Zn, for each h E H there exists x E Zn such that d(h, x) < 1/n, so U°_IZ, is dense. If H is not separable, then this implies that Zn is uncountable for some n. Now we prove that An is separable for all n. Indeed, assuming that it is not separable, there is an uncountable discrete closed subset Zn in An; this set is dicrete in An+1 and is not separable, so there exists an uncountable,
discrete, closed subset Zn+l in Zn. Thus, we obtain a sequence Z,n such that Z,,,, 3 Z,,,, for ml < m2, and Z,n is an uncountable, discrete, closed subset of A,,,,. Since K is compact, the set Z;,, of accumulation points of Z,n is nonempty; therefore n°O=nZm 0. On the other hand, Z;,, nA,n = 0 since Z.. is discrete and closed in An, and therefore 0, and which is a contradiction. Fix k and consider the set A n n Ak (k < n), which is a separable metric space and therefore has a countable basis {Bn,,,, : m E w}. Let Gn,m be an open set in Ak such that Bn,m = Gn,m n An. The family {Gn,m : n > k, m E w} is a subbasis for a Hausdorff topology on Ak, which is coarser than the compact topology of Ak; therefore, these two topologies are equal. This means that Ak is a second-countable, compact topological space. Let {fn,k : n E w} be a family of continuous functions on Ak that separates the points. Let Fn,k be a continuous function on K for which Fn,klAk = fn,k
(Such functions Fn,k exist by the Tietze theorem.) The functions Fn,k separate the points of K; therefore the metric defined by these functions induces the topology of K.
3.10 TOPOLOGY
547
Problem T.19. Let U denote the set If E C[0,1] : If (x) I < 1 for all x E [0, 1]}. Prove that there is no topology on C[0,1] that, together with the linear structure of C[0, 1], makes C[0,1] into a topological vector space in which the set U is compact. Solution. We assume that topological vector spaces are Hausdorff spaces. Arguing by contradiction, assume that C[0,1] is a topological vector space in which U is compact. Then U is closed since the topology is Hausdorff. For each f E C[0, 1] and 6 > 0, the set Uf,6 = {g E C[0,1] : lg(x) - f (x) 1 < b for all x E [0, 1]}
is also compact and closed. Define the functions fn, gn E C[0,1] for each natural number n > 2 as
if0<x<2-n,
10 f,, (x) =
nx + 1 - 2
if 2 - n < x <
1
if1<x<1,
10 gn(x) = nx - 2 1
2,
if0<x< 2, if 2 < x < 2 + n, if
2+n
<x<1.
Put Kn = {g E C[0,1] : gn(x) < g(x) < fn(x) for all x E C[0,1]} (n > 2). fl Then K2 D K3 3 , and Kn is compact and closed since Kn = Ugn_1,1. So nn'=2Kn # 0. Let f E nn'=_2Kn. Then f (x) = 0 for x < 1/2, and f (x) = 1 for x > 1/2, which is impossible, since f is continuous. This contradiction shows that U cannot be compact. Problem T.20. Let 4 be a family of real functions defined on a set X such that k o h E 4 whenever fi c 4 (i E I) and h : X - 1[81 is defined by the formula h(x)i = fi(x), and (1) k: h(X) ---> 1[8 is continuous with respect to the topology inherited from the product topology of R1. Show that f = sup{gj : j E J, gj E implies f E 4. Does this statement m E M, hn,, E remain true if (1) is replaced with the following condition? (2) k: h(X) -> 1[8 is continuous on the closure of h(X) in the product topology.
Solution. We may obviously assume that J and M are disjoint. Define p : X -+ JJUM by p(x)j = gj (x), for j E J, and p(x),,+. = h,,, (x), form E M. Then, for ally E p(X), we have sup{yj : j E J} = inf{y,n : m E M}. Define k : p(X) -> JR by k(y) = sup{yj : j E J} = inf{ym : m E M}. Then f = k o p, so, in view of (1), it suffices to show that k is continuous on p(X). Let y E p(X) and E > 0. We shall define a neighborhood S of y in
p(X) such that Ik(z) - k(y)l < E for all z E S. Choose jo c J with
548
3. SOLUTIONS TO THE PROBLEMS
gjo(y) > k(y) - (e/2) = sup{gj(y) : j E J} - (E/2), and choose mo E M with h,,,.o(y) < k(y) + (E/2) = inf{hm(y) : m c M} + (e/2). Such jo and mo exist by the definition of supremum and infimum. Put
S = {z ep(X) : xjo -Yjo1 < 2 and Izmo - ymoI < 2 Then, for z E S, we have
k(z)=sup{zj:jEJ}>zjo>yjo-2>k(y)-e and
that is, k(z) - k(y)I < E.
Therefore, for every y E p(X) and e > 0, there exists a neighborhood S
of y such that Ik(z) - k(y)I < e for all z E S, that is, k is continuous on p(X). This solves the first part of the problem. If condition (1) is replaced with (2), then the statement is false. Indeed, let X = N be the set of natural numbers, and let 4) consist of all functions f : N -p ]R such that the sequence { f (n)}n 1 is convergent. We claim that this is a counterexample. Suppose that fi E 4) (i E I), that h : N R is defined by h(n)i = fi(n)
(i E I), and that k : h(X) -> R is continuous. Let b be the element of R1 such that bi = lim fi. Then, by the definition of h, h(n)i -+ bi for all i E I; therefore, h(n) -4 b in the product topology. This implies that b is contained in the closure of h(X) since it is the limit of a sequence in h(X). By continuity of k on h(X), we have that k(h(n)) -+ k(limh(n)) = k(b), that is, the sequence k(h(n)) is convergent, and therefore k o h E This shows that the hypotheses of the problem are satisfied. Let J = M = N; let gj (n) = 1, for n < j and n even, gj (n) = 0 otherwise; and let hm(n) = 0, for n < m and n odd, hm(n) = 1 otherwise. These functions are indeed in (P, since gj(n) --+ 0 and hm(n) -+ 1 for all j E J and
m E M. On the other hand, sup{gj(n) : j E J} = inf{h,.. (n) : m E M} equals 1 if n is even and equals 0 if n is odd. This sequence is not convergent;
therefore f = sup{gj(n) : j E J} = inf{h,,,(n) : m E M} does not belong to
Problem T.21. Characterize the sets A C R for which
A+B={a+b:aeA, bEB} is nowhere-dense whenever B C R is a nowhere dense set.
Solution. We prove that the sets with this property are the bounded sets with countable closure. More precisely, we show that for a set A C R the following are equivalent:
3.10 TOPOLOGY
549
(i) If B C R is nowhere dense, then so is A + B; (ii) For any sequence an (n = 1, 2, ...) of positive numbers, there exists a finite family of intervals II (j = 1, ... , k) that covers A and such that the length of Ij is II, I = aj (j = 1, 2, ... , k); (iii) A is bounded and its closure is countable. (i) = (ii): Suppose that (ii) does not hold. Then there exists a sequence consisting of positive numbers an (n = 1, 2, ...) such that for any finite family of intervals Ij, jIj I = aj (j = 1,2,... , k) we have A ¢ Uj=1Ij. Using this, we shall construct a nowhere-dense set B such that A + B contains all rational numbers, which contradicts (i). Arrange the rational numbers into a sequence rn (n = 1, 2.... ). Now we define the numbers xn and yn by induction on n. Let x1 E A be arbitrary, and y1 = r1 - x1. Suppose that k > 1 and the numbers y1, y2, , yk-1 have already been defined. Then, by our assumption, the intervals
h- (rk - yJ
2
,
rk - yJ +
2I' j
=1,2,...,k-1
do not cover A. Therefore, we can choose an element xk E A \ Finally, put Yk = rk - xk. Then, with B = {yk : k E N}, the set A + B contains all rational numbers since rk = xk + yk E A + B. We show that
every point of B is isolated. Indeed, for j < k, we have xj (rk - yj aj /2, rk - yj + aj /2). Therefore, yk = rk - xk V (yj - a3/2, yj + a3/2). This means that the set B contains at most j points in the neighborhood of radius aj/2 of the point yj, and thus yj is an isolated point of B. This implies that B is nowhere dense. (ii) (iii): If (ii) is true; then A is obviously bounded. Arguing by contradiction, suppose that (iii) does not hold. Then the closure of A is
uncountable and, by the Cantor-Bendixson theorem, contains a nonempty,
bounded, and perfect set P. Let us call a sequence (a1i a2, ...) of positive numbers insufficient if for any family of closed intervals Ij, I3 I = a3 (j = 1, 2, ... , k), we have P ¢ In order to arrive at a contradiction, by induction we shall define an infinite sequence of positive numbers (a1i a2, ...) whose all finite initial segments are insufficient. Let a1 be a
positive number that is smaller than the diameter of P; then (a1) is obviously insufficient. Suppose that n > 1 and we have already defined an insufficient sequence (a1i a2.... , an). We show that there exists a number an+1 such that (a1i a2, ... , an+1) is insufficient. Suppose this is not true; then for every k E N and an+1 = 1/k there exists a sequence of intervals Ij,k, JIj,k = aj (j = 1, 2, ... , n+ 1) such that P C U +1 Ij,k holds. We may assume that the intervals Ii,k all meet the set P, and thus they are all contained in a bounded set. By passing to a suitable subsequence, we may also assume that for each fixed j = 1, 2, ... , n + 1, the sequence of intervals Ij,k converges to an interval I j when k --f oo. Then II; = aj, for j = 1, 2, ... , n, and In+1 consists of a single point. Since (a1, a2, ... , an) is insufficient, the set P is not contained in U 1I3 . Moreover, being perfect, P has infinitely many points in the complement of U 1II. So we can choose a point
550
3. SOLUTIONS TO THE PROBLEMS
x E P \ U±i
But then, for sufficiently large k, x 0 U +1 Ij,k, which . contradicts the choice of the intervals Ii,k. This proves that the sequence (ai, a2i ... ,1/k) is insufficient, completing the induction. (iii) = (i): Suppose that, to the contrary of (i), with a nowhere-dense set B, the set A + B is dense in some interval I. Let A0 denote the closure of A; then A0 + B is also dense in I. Define the sets A,,, for all countable ordinals as follows: If /3 > 0 is an ordinal such that A,, has already been defined for all a < 3, then let Ap = n,<,3A0, if 0 is a limit ordinal, and
let A0 be the set of accumulation points of A,, if 0 = a + 1. Since A0 is countable, there exists a countable ordinal ry such that A., = 0. Then obviously A.1 + B = 0 and A., + B is not dense in I. Let ,3 denote the smallest ordinal for which Ap + B is not dense in I; then ,3 > 0. Let J be a subinterval of I with (A0 + B) n J = 0, and let 0 < E < IJ1/3. Let U(H, S) denote the neighborhood of radius S of the set H. It is easy to see that U(H, S) + B C U(H + B, S) holds for every set H and b > 0. Thus, U(A0, e) + B C U(A0 + B, E), and therefore U(A0, E) + B does not meet the middle third K of J. Suppose that 0 is a limit ordinal. Since {A,,, : a < 3} is a nested sequence
of compact sets whose intersection is A0, there exists a < 0 with A, C U(A0, E). Then A,, + B does not meet K. But this is impossible, since a < ,3 and A,,, + B is dense in I. Finally, suppose that 3 = a + 1. Then V = A, \ U(Ao, E) is a finite set, since all accumulation points of A,, are in A0. Then the set V + B is nowhere dense. Therefore, from (A,, + B) fl K = (V + B) fl K, it follows that (A, + B) fl K is nowhere dense. But again this contradicts that a < /3 and that A,, + B is dense in I. This completes the proof. Problem T.22. Prove that if all subspaces of a Hausdorff space X are or-compact, then X is countable.
Solution. Since X itself is a union of countably many compact sets, it is sufficient to prove the statement for compact spaces X. Suppose that X has no subset that is dense in itself. Then there is a well-ordering on X in which all initial segments are open. Indeed, suppose that the points pp E X have already been defined for all ,3 < a and that
Xa = {pp : /3 < a} # X. Then there exists a point pa E X \ X0. that is not an accumulation point of X \ X0,, and so p0. E G. C_ X. U {p.} with a suitable open neighborhood G0, of p,,. This recursion defines a wellordering of X, and UQ<0,GQ C X. = U0<0,{p0} C_ Up<0,Gp shows that X4 = Up<0.Gp is an open set for each a. An initial segment of order type
wi in X cannot be or-compact since a compact subset in X must have a greatest element. Therefore, X is countable. Thus, it suffices to prove that X does not contain a subset dense in itself. Arguing by contradiction, assume that there exists such a subset Y. Then Y is dense in itself and closed. We define a continuous function f : Y -+ [0, 1] as follows. Since Y is a compact Hausdorff space, for any two distinct points p and q there exist open sets Gp, Gq and closed sets FP,
3.10 TOPOLOGY
551
Fgsuch that pEGp9Fp,gEGg9Fq,andFpnFq=0. So, for all finite 0-1 sequences lei}, we can define closed sets AE1i... En C Y so that AEI,...,e.,o n AEI,...,En,l =0
and
AE1....,e, ,O U AEl ...,e, ,l C A,, ---,En
hold for all natural numbers n and el, ... , en E 10, 11. For any infinite 0-1 sequence e1,E2,...,the set 00
A(El, E2....) = n A,
62,--En
n=1
is nonempty and closed. If x E A(E1, E2.... ),then put °O
f (x)
1
Ei 2i
i=1
The function f maps A(El, E2, ... ), which is a closed set since it is an intersection of closed sets, continuously onto the interval [0, 1]. Then f extends continuously over V with Im f = [0, 1]. The interval [0, 1] contains 2x° or-compact sets (that is, Fe-sets), since the number of closed sets in [0, 1] is 2x°. Therefore, there exists a set Z C [0, 1] that is not or-compact; then its inverse image f -1(Z) C V cannot be a-compact, which contradicts the hypothesis of the problem. This proves that X contains no subset that is dense in itself.
3. SOLUTIONS TO THE PROBLEMS
552
3.11 SET THEORY Problem K.1. Does there exist a function f (x, y) of two real variables that takes natural numbers as its values and for which f (x, y) = f (y, z) implies x = y = z ?
Solution 1. First, we show that it is sufficient to construct for each real number c, two sets Ac and Bc of natural numbers that satisfy the following conditions: (1) 1 V Ac, 1 V Bc for all c,
(2) A,nB,=O forallc, (3) A.nBd54 0forc#d. Indeed, put f (c, c) = 1, and for c # d, let f (c, d) be an arbitrary element of the nonempty set Ac n Bd. Now, if
f(x,y)=f(y,z)=m01, then
mEAxnBy
and
mEAynB,,
that is,
mEA,nB.nAynBZ cAynBy=O, which is a contradiction. But m = 1 is only possible when x = y = z.
In order to construct Ac and B,, map the set R of rational numbers bijectively onto the set N = 12,3.... }; let n(r) denote the image of r E R. Assign to each real number c a sequence {rk(c)}- 1 that converges to c and consists of rational numbers different from c. Let A, = {n(rk(c)) : k = 1, 2, ... } c N,
B,=N-Ac. It is easy to see that these sets Ac and Bc satisfy conditions (1), (2), and (3). Thus, we have constructed a function with the required property.
Solution 2. Arrange the set of rational numbers into a sequence r1, r2, . Define the function f (x, y) the following way: If x = y, put f (x, y) = 1; If x < y, put f (x, y) = 2i, where i is smallest possible with x < ri < y; If x > y, put f (x, y) = 2i+1, where i is smallest possible with x > ri > y. We show that f (x, y) is as required. It is obviously defined on the reals, and its range is the set of natural numbers. Suppose that
f(x,y)=f(y,z)=m. If m = 1, then x = y = z is the only possibility. If m > 1 and m = 2i, then
x
and
y
3.11 SET THEORY
553
which is a contradiction. Similarly, if m > 1 and m = 2i + 1, then
x>ri>y
and
y>ri>z,
which is again a contradiction. Therefore, x = y = z follows.
Solution 3. We will decompose the plane into a countable family of pairwise disjoint subsets with the following property: if one of the lines parallel to the coordinate axes through the point (y, y) intersects one of the subsets, then the other does not, except when the subset is the line x = y, which will be one of the subsets. It suffices to construct such a partition of the open half-plane below the line x = y, because by reflecting in the line x = y and adding the line x = y, we obtain the required partition of the whole plane.
Put
Ao={(x,y):x>0, y<0},
Ak={(x,y):x>k, k-1
An = { (x, y) : An k =
{
2k-1 2n
2k
< x < 2n ,
2k-2 2n
-2k+1 <X< -2k+2 )n , 2n
2k-1 2n
,
-2k -2k + 11 2n < y C 2n j
where k, n = 1, 2, .... It is obvious that the union of these sets is the open half-plane below the line x = y. This defines a partition of the whole plane, as we said above. Label the sets of this partition by the natural numbers, and assign to the point (x, y) the label of the set containing (x, y) as the value of the function f (x, y). If x, y, z are real numbers with f (x, y) = f (y, z), then the points (x, y) and (y, z) are in the same subsets, but then (x, y) (and (y, z)) is on the line through the point (y, y) parallel to the x-axis (y-axis, respectively), and any of the subsets can only meet one of these, except for the subset
{(x,y):x=y}, but then x=y=z. Remarks.
1. Obviously, the existence of such a function is a question of cardinality. In general, it makes sense to ask whether there exists a function f (x, y)
defined on a set A of cardinality m that takes its values in a set B of cardinality n (both m and n are infinite) and for which f (x, y) = f (y, z) implies x = y = z. J. Gerlits, L. Lovasz, L. Posa, and M. Simonovits proved that such a function exists if and only if m < 2°. Here we present the proof by Posa.
3. SOLUTIONS TO THE PROBLEMS
554
Let m < 2". It suffices to give the construction for a set A of cardinality 2". Let a be the smallest ordinal of cardinality n. We may assume that A is the set of sequences of the numbers 0 and 1 of order type a. Define the function f (x, y) as follows:
If x = y, put f (x, y) =0; If x # y, then let 13 be the smallest ordinal where the elements in x and y are different, let j be the 13th element of x (j = 0 or 1), and put f (x, y) = (13, j). The range off is the set of the symbol 0 and the ordered pairs (13, j), where 13 < a and j = 0 or 1. Therefore, it has cardinality n and can be mapped bijectively onto B. f (x, y) = f (y, z) = (13, j) means that all elements before 13 agree both
in x and y, and in y and z, and the 13th element is j in x and y. But then the 13th elements of x and y are equal, which is a contradiction. Therefore, f (x, y) = f (y, z) = 0, which implies that x = y = z. (This part of the statement is also proved by Bollobas and Juhasz.)
Suppose that m > 2", and there exists a function required in the problem. Consider the range of f (x, y) when x is kept fixed. This is a subset of B, and since the cardinality of all possible x's is greater than the cardinality of the set of all subsets of B, for some x1 $ x2 the range of f (xii y) equals that of f (x2, y). So, the value f (x1, x2) is contained in the range of f (x2i y), that is, for some yo, f (x1, x2) = f (x2i yo), which is a contradiction. 2. Bela Bollobas and Miklos Simonovits notice that there exists a function F(xl,... , xn) defined on the real numbers that takes its values in the natural numbers, and for which F(a, x2i ... , xn) = F(yi, a, y3, ... , yn) = ... = F(u1,... , un-1, a)
implies a = xi = yi =
= ui (i = 1, 2, ... , n). For example, n
F(x1i... , xn) _ 11
pj(ijxi,xi)
i,j=1
is such a function, where the pij (i, j = 1, ... , n) are different primes, and f (x, y) is the function defined in either solution of the problem.
Problem 1t.2. Prove that there exists an ordered set in which every uncountable subset contains an uncountable, well-ordered subset and that cannot be represented as a union of a countable family of well-ordered subsets. Solution. Let R be the set of all limit ordinals less than w1i and for a E R, let fa(n) be a monotone increasing sequence (of order type w) of ordinals converging to a. Order R as follows: for a,13 E R, a -< 13 if f, (n) < fp(n) holds for the first n with fa(n) 54 f1i(n).
3.11 SET THEORY
555
Any uncountable subset X of R contains a well-ordered subset of order type w1 with respect to the ordering -. Indeed, let
XIn = { (ao, al, ... , an) : there exists a E X, such that f, ,(m) = a(m) for all m < n}. For n sufficiently large, X In is uncountable, since for large enough n the ordinal
ryn=sup{fa(n): aEX} equals w1, since, by f,, (n) - a,
sup{ryn:n<w}=sup{a: aEX}=w1. Order XIn lexicographically, that is,
(ao,...,an) -<' (00,
On)
if am <,3m for the smallest m with am # /3m. Then X In is well-ordered and, if n is large enough for X In to be uncountable, it contains a wellordered subset of order type w1. Let X' be a subset of X such that for any
(ao,...,an) E XIn, there exists a unique a E X' with f.(0) = ao,...,fa(n) = an.
Then the ordered set (X', -<) is isorr. )rphic to (X In, -<'). Since we have seen that the latter contains a well-ordered subset of order type w1i the same is true of the former. This is what we wanted to prove. Now we show that R is not a union of a countable family of well-ordered subsets. First we make a digression. A function that maps a set of ordinals into the ordinals is called regressive if f holds for all ordinals # 0 in the domain of f . If the domain of f is a subset of w1 and the set
{ : f() < Al is not cofinal with Wl for any p < w1, then f is called divergent. X C wl is called thin if there exists a regressive divergent function defined on X; otherwise, it is called stationary. The set wl is stationary. Indeed, assuming that f is a regressive function on w1, the sequence
S'f(01 f(f(0),... , being a descending sequence of ordinals, can contain only a finite number of elements different from zero. Therefore,
W1 = U Xn n<w
556
3. SOLUTIONS TO THE PROBLEMS
where
Xo = {0}, and
X.+1
= {e : f (e) E Xn}. Assuming further that f is divergent, induction on n shows that sup Xn < w1 for all n, which obviously is a contradiction. From this it easily follows that the set R of limit ordinals less than w1 is also stationary. It quickly follows from the definition that the union of a countable family
of thin sets is also thin. Further, if X is stationary and f is a regressive function on X, then for some p < w1 the set
is stationary (this is a special case of Fodor's theorem). Indeed, assume that all Xµ are thin, and let fµ be a regressive divergent function defined on X. Then the function g defined on X by max(p, f,, (C))
(C E Xµ, p < wi)
is regressive. It is divergent, too, because for all v < w1 the set
{C:g(c)
is not cofinal with wl, since neither one of the summands in the righthand side is, because the functions fµ are divergent. This contradicts the assumption that X is stationary, which proves our statement. Let X C R be stationary. We show that X is not well ordered with respect to the ordering -<. Put Y_1 = X and
Yn={aEYn_1 : fa(n)=6n}, where 6n is the smallest ordinal for which Yn is stationary (such 6n exists by the Fodor theorem proved above).
Put
X'=X- UYn, n<W
where
Y,'={aEYn-1:fa(n)<6n}. By the choice of 6n, Y,' is not stationary, and neither is X - X'. Put 6 = sup{6n : n < w}.
It is obvious that 6 < w1. Let a E X' be greater than 6. (X' is stationary and so it is cofinal with w1.) Since fa(n) -p a, for some n < w we have f, (n) > 6 > 6n,
3.11 SET THEORY
557
so f, (m) = 6,,,, cannot hold for all m. It follows from a E X' that fa (m) >
8,,, for the smallest m with f, (m) # 6m. Thus, for this m, a is greater than all elements of Y,,, with respect to the ordering -<. Put X0 = X, ao = a, and Xl = Y,,,,. Since Y,,,, is stationary, we can repeat the above argument with Xl instead of X0. We obtain a1 and X2,
then a2 and X3, and so on. Since ao >- a1 >- a2 >- ..., X is not wellordered with respect to the ordering -<, and this is what we wanted to prove.
Now, if R = Un< R,,, then some R is stationary, and this R, is not well-ordered with respect to -<. This proves the second statement of the problem.
Remark. Laszlo Babai showed that for each cardinal number is > w, there exists an ordered set B of cardinality is such that all subsets of cardinality r. contain a well-ordered subset of cardinality K and B is not a union of a family of cardinality less than K of well-ordered subsets.
Problem R3. Let < be a reflexive, antisymmetric relation on a finite set A. Show that this relation can be extended to an appropriate finite superset B of A such that < on B remains reflexive, antisymmetric, and any two elements of B have a least upper bound as well as a greatest lower bound. (The relation < is extended to B if for x, y E A, x < y holds in A if and only if it holds in B.)
Solution. B will consist of all the elements and subsets of A without identifying the elements with the corresponding singletons. In what follows, lowercase letters denote elements, whine capitals mean subsets. Define < as follows:
a < b,
as given,
a
Problem 1t.4. Let .F be a family of subsets of a ground set X such that UFE.FF = X, and (a) if A, B E.F, then A U B C C for some C E .F;
(b) if A E F (n = 0,1, ... ), B E F, and Ao C Al C .... then, for some
k>0,A,,,nB=AknBforalln>k.
3. SOLUTIONS TO THE PROBLEMS
558
Show that there exist pairwise disjoint sets X., (7 E F), with X = U{X.y : 7 E F}, such that every X y is contained in some member of .F, and every element of.F is contained in the union of finitely many X.y's.
Solution. Let < be a well-ordering of F. Let F be the set of all finite subsets of F. Well order F as follows:
71,72EF,7154 72,71={A1i...,A,}, 72={B1,...IBm}1
An <... 72 if Ai = Bi for 1 < i < m. If, on the other hand, i < m is the least number with Ai 0 Bi then put 7i < 72 or 71 > 72 if Ai < Bi or Ai > Bi. A straightforward checking shows that this well orders F. We notice that 71 < 72 implies 71 < 72. For 7 E F, we define Y.y E F as follows. If y = {A}, put Y.y = A. Assume that Y6 is defined for b < 7. Then let Yy be a member of F, covering every Yp (,Q < 7). This is possible by (a). Now put X.y = Yy \ U6<.yY6. The only nontrivial thing to prove is that
every A E F is covered by finitely many Xp. We prove by transfinite induction on -y that Yy is covered by finitely many X6's. If this first fails
for 7, there exists /31 < 02 < ... < 7 such that Y.y n Xp # 0. Let Y y = {Al,... , Ak}, A ,>. . .> Ak. For every n there is a 1 < j < k such that
A1i...,Aj_1E/n,
A.7Oi3
.
We can assume, by shrinking, that j is the same for every n. Put ijn = max(01, ... , Qn) < 7. As 7 is minimal, Yp meets finitely many X. As Yn, C Yn2 C
..., by (b) there is an m such that Y,,m nYy = Ynm+, nYy = ....
Xnm+: n Yy # 0 (i = 0, 1, ...) that is, Y1m meets X'7_' X,7-+,, ... , a contradiction. so Y,,m
Ynm+: n Yy
Problem R.5. Show that there exists a tournament (T, -4) of cardinality N1 containing no transitive subtournament of size N1. (A structure
(T, -) is a tournament if - is a binary, irreflexive, asymmetric, and trichotomic relation. The tournament (T, -4) is transitive if - is transitive, that is, if it orders T.) Solution. We use the existence of,a Specker type, that is, an ordered set (A, <) of size ail, not containing subsets similar to w1, wi, any uncountable subset of the reals. See (P. Erdos, A. Hajnal, A. Mate, R. Rado, Combinatorial set theory: Partition Relations for Cardinals, Akademiai Kiado, Budapest, 1984, p. 326.) Enumerate A as {a(a) : a < w1 }, and let {x(a) : a < w1} be different real numbers in [0, 1]. If a < /3, put a +- ,Q iff either x(a) < x(/3) and a(a) < a(/3) or x(a) > x(/3) and a(a) > a(,3).
3.11 SET THEORY
559
Let B C A be uncountable. Let X = {a < w1: a(a) E B}. Claim 1. There is an a E X such that {/3 E X : a(a) < a(0)
and
x(a) < x(,3)}
is uncountable.
Proof. For a E B, let f (a) be the least t E [0,11 such that x(3) < t for all but countably many 3 E X, a(,3) > a. Since f is a non-increasing real-valued function on (A, <), it can only have countably many different values; otherwise, there would be an uncountable subset of A, similar to a set of reals. Hence f is constant on an uncountable B0 C B. Define X0 analogously to X. We can choose ao E X0 such that {,Q E Xo: x(/3) > x(ao)} is uncountable; otherwise (A, <) would contain a subset of type wi.
Now we can choose a E Xo such that a(a) > a(ao) and x(a) < x(ao). Since g(a(a)) = g(a(ao)) > x(a), there are uncountably many /3 E X such
that a(3) > a(a) and x(Q) > x(a).
Claim 2. There are uncountable X0, X1 C X such that if a E X0, Q E X1, then a(a) < a(,3) and x(a) < x(/3).
Proof. Let U be the set of those a E X such that {/3 E X : a(a) < a(0)
and x(a) < x(,0)}
is countable, and let L be the set of those a E X such that {,Q E X : a(,3) < a(a)
and x(,3) < x(a)}
is countable. If U or L is uncountable, we get a contradiction by Claim 1, so we can select a E X \ U \ L, and put Xo = {,Q E X : a(,3) < a(a)
X1 = {/3 E X : a(a) < a(/3)
and x(/3) < x(a)}, and x(a) < x(,3)} .
By an application of Claim 2 to X0 and the Specker type (A, >), one can find Yo, Y1 C_ Xo such that a E Yo, ,0 E Y1 imply a(a) < a(,3), and x(a) > x(Q). Now select a E Yo, 3 E X1, -y E Y1 with a < Q < y. Then a <- /3 +- y and -y +- a, so our tournament is not transitive on B. Remark. The result in the problem was first proved by R. Laver.
Problem R.6. Assume that R, a recursive, binary relation on N (the set of natural numbers), orders N into type w. Show that if f (n) is the nth element of this order, then f is not necessarily recursive.
Solution. We use the following well-known facts. There is a set A C N that is recursively enumerable, that is, the range of a recursive function, but
560
3. SOLUTIONS TO THE PROBLEMS
is not recursive, namely, its characteristic function is not recursive. For every infinite r.e. set A there is a recursive function enumerating A's elements in a one-to-one manner. An infinite set possesses an increasing such enumeration if and only if it is recursive. Take a (necessarily infinite) r.e. but not recursive set A, and a function g enumerating A without repetition. Put aRb if g(a) < g(b). Clearly, R is recursive and orders N into type w. If f (n)=the nth element by this order, and f is recursive, then the recursive h(n) = g(f (n)) would enumerate A's elements in increasing order, which is impossible.
Problem R.7. Let H be the class of all graphs with at most 2K0 vertices not containing a complete subgraph of size 1Z1. Show that there is no graph
H E H such that every graph in H is a subgraph of H.
Solution. We have to show that if (V, G), a graph with IVI = 2s0, does not contain a complete graph on R1 vertices, then there is a graph (W, H) with these properties such that (W, H) cannot be embedded into (V, G). Let W be the set of those functions injecting a countable ordinal (possibly 0 # 0) into a complete subgraph of G. To define H, join two such functions if one extends the other. Clearly, Ja->VI
JWJ= a<wl
First, we show that (W, H) does not contain a complete R1-gon. Assume
that If,,: a < wl } are pairwise joined, and that the ordinals Dom (fe) are in increasing order. By the construction of H, for,3 < a, f,, extends fQ, so the union of the fa's gives a function f : w1 --+ V onto a complete R1-gon in (V, G), a contradiction. Next we prove that (W, H) may not be embedded into (V, G). Assume that h: W -4V is such an embedding. By transfinite recursion on a < w1, we define fa : a -4V, fc. E W, and x,, = h(fa) E V in such a way that fa extends fp for i < a. Put fo = 0, fo E W. If fp (i3 < a) are defined, then
{xo:,3
the requirements. But then {x,,: a < W11 is a complete subgraph in G, a contradiction. Remark. This is an unpublished result of R. Laver. Problem N.8. For which cardinalities ,c do antimetric spaces of cardinality is exist? (X, o) is called an antimetric space if X is a nonempty set, o : X2 -+ [0, oo) is a symmetric map, o(x, y) = 0 holds iff x = y, and for any three-element subset {a1, a2, a3} of X
o(alf,a2f) + o(a2f,a3f) < o(aif,a3f) holds for some permutation f of {1, 2, 3}.
3.11 SET THEORY
561
Solution. For 0 < ,c < 2H0. First, if X C_ R is nonempty, then (X, o) is antimetric, where o(x, y) = (x - y)2 and clearly I X I can take any value in the given interval. For the other direction, let (X, o) be antimetric and IXI > 2K0. Color the complete graph on X by the integers as follows. Let {x, y} get color k if 21c < o(x, y) < 2k+1 By the Erdos-Rado theorem (see P. Erdds, A. Hajnal, A. Mate, R. Rado, Combinatorial Set Theory: Partition Relations for
Cardinals, Akademiai Kiado, Budapest, 1984, p. 98), there exist x, y, z such that {x, y}, {x, z}, {y, z} get the same color. But then, x, y, z do not meet the requirement on antimetricity.
Problem 3t.9. If (A, <) is a partially ordered set, its dimension, dim (A, <), is the least cardinal K such that there exist K total orderings { Rio, then there exist disjoint Ao, Al C A with dim(Ao, <), dim(Al, <) > No. Solution. Assume indirectly that (A, <) is a counterexample of minimal cardinality A. Without loss of generality, A = A. Put
I={BC A:dim(B,<)
Claim 1. If B,,, E I (n = 0,1, ...) and B is such that every pair of B is covered by some Bn, then B E I. Proof. Straightforward from the definition.
Claim 2. There are Bo, B1 E I such that Bo n B1 = 0 and Bo U B1 ¢ I.
Proof. If not, then I is a a-complete primideal containing every subset of cardinality < A. For a < A, let
Put
I1={CC B1:BoUCEI}. Then B1 ¢ Il and every subset of B1 of size < is is in I,. Repeating the argument of Claim 2, we get that there exist Co, C1 E Il such that Co n C1 = 0 and Bo U Co, Bo U Cl ¢ I. Repeating the above argument for Co, Bo this time, one gets Do, D1 C Bo, Do n Dl = 0 such that Do U Co, D1 U Co E I, Do U D1 U Co ¢ I. As every pair of Do U Dl U C1 is contained either in Bo or in Do U C1 or in D1 U C1, by Claim 1, either Do U C1 or D1 U C1 is not in I. In the former case, Do U C1 and D1 U Co are two disjoint sets not in I, and we conclude similarly in the other case, too.
3. SOLUTIONS TO THE PROBLEMS
562
Problem 12.10. A binary relation -< is called a quasi-order if it is reflexive and transitive. The infimum of the quasi-order (Q, -<) is the greatest subset J C_ Q such that (i) for every B E Q there is an A E J with A - B, and (ii) A -< B, A, B E J imply B -< A. Let X be a finite, nonempty alphabet, let X* be the set of all finite
words from X, and let P be the set of infinite subsets of X*. For A, B E P, let A - B if every element of A is a (connected) subword of some element of B. Show that (P, -<) has an infimum, and characterize its elements.
Solution. As follows, subword will always mean connected subword. Let a < b denote that a is a subword of b. Call A E P minimal, if for all w E A, all but finitely many subwords of the elements of A contain w, as subword. Claim 1. If A is minimal, B
A, then A
B.
Proof. Assume that w E A and k is so large that every subword in A longer than k, as B -< A, is a subword, so w < b. Claim 2. If A E P is not minimal, then there exists a B E P such that B
Abut A.AB. Proof. Let w be a subword of A such that
H={h: there is an aEA,h
Proof. Such a B is given by the following algorithm. Step 1: Let X = {al, ... , an}. If, among the subwords of A, infinitely many do not contain a1 i let Hl be their set. H1 E P, H1 -< A, and it does not contain al. Then get successively H2, H3, ... using a2, a3, .... If Hn_1 is defined, it may only contain an, so it is minimal. We can therefore assume the existence of Hi_1 such that all but finitely many subwords of Hi_1 contain ai. Put b1 = ai, B1 = Hi_1 -< A, and there follows Step 2. Here Step k: If bk_1i Bk_1 are given, and b1 < ... < bk_1i Bk_1 -< B1 - A are given, and all but fi-itely many subwords of Bt contain bt, consider the word bk_1a1. If infinitely many subwords of Bk_1 omit it, let Hi -1 be this set. bk_1a1 therefore is not a subword of Hi-1. Repeating this to bk_1a2i we get H2 -1, etc. If we can reach Hn=i, then there exists a natural number in, such that in every subword of length 2m a bk_1 can be found in the first m positions, but it must be followed by an. So we can select Hk-1 as Bk and bk_1ai as bk. By this process, we get b1 < b2 < .... B1 >- B2 ... . Put L = {b1i b2, ... }. Then L A and L is minimal. In fact, for bi c L , E Bi, so only finitely many subwords of them omit bi. bi+i, bi+2, We get that the minimal members constitute the infimum. Claim 3 gives (i), Claim 1 (ii), and the maximality holds as if A is nonminimal, B -< A
3.11 SET THEORY
is such that A 7k B, C is minimal, C - B, then C -< A, A
563
C, so the
addition of C would violate (ii).
Problem N.11. Define a partial order on all functions f : R -* R by the relation f -< g if f (x) < g(x) for all x E R. Show that this partially ordered set contains a totally ordered subset of size greater than 2s0 but that the latter subset cannot be well-ordered.
Solution. For the first part of the problem, it suffices to find a family of more than 20 subsets of R totally ordered by c, as then the characteristic function will do the job. In order to show that if is is an infinite cardinal, then there are #c+ subsets of a set of size K, ordered by C, let A be the least
cardinal such that 2A > ic. By Cantor's theorem, .\ exists and is < K. If we find a totally ordered set of size f+, with a dense subset of cardinality < #c, we are done by the Dedekind cuts. For such a set, take the .\ -> {0, 1}
functions, ordered lexicographically. A dense subset is formed by those functions that take 0 from a place onward. Their number is
1: 21`'
For the second part, assume that f0, < fp for a < 0 < (2"0)+. Let x0, E R satisfy f,, (x,,) < f,,+1(xa), for (2k0)+ many a, xa = x, and so the f,, (x) are (2'0) + different elements of IR, which is impossible.
Index of Names A
C
Abel, N. H., 489 Aczel, J., 192 Adam, A., 16 Adian, S. I., 82 Ajtai, M., 30
Cayley, A., 421 Clifford, A. H., 85 Corradi, K., 10 Coxeter, H. S. M., 91 Csakany, B., 8, 22, 28, 46 Csaszar, A., 2, 4, 7, 12, 13, 18, 21, 30, 42 Csikos, B., vii, 46, 52 Csorgo, M., 443 Csorgo, S., 35, 42, 48 Czach, L., 12 Czedli, G., 46, 51 Czipszer, J., 2
B Babai, L., 25, 32, 50, 51, 282, 491, 529, 557 Baer, R., 63, 81 Bajmoczi, E., 51 Balogh, A., 39 Balogh, Z., 32, 33, 51 Bara, T., 51 Beck, J., 51 Beleznay, F., 52 Benczur, A., 53 Berlekamp, E. R., 374 Bernstein, I. N., 332 Bernstein, S., 158 Birkas, Gy., 53 Biro, A., 53 Bod6, Z., 52 Bognar, J., vii Bognar, M., 13, 20, 21, 27 Bognarne, K., 425 Bohus, G., 52 Bolla, M., vi Bollobas, B., 50, 263, 518, 554 Boltjainskii, V. I., 184 Borges, R., 3 Boros, E., 51 Bosznay, A., 32 Brindza, B., 298 Buczolich, Z., 52 Burnside, W., 96, 118
D
Daroczy, Z., 12, 20, 25, 38, 192, 513
Deak, J., 51 Denes, J., 33 Deny, J., 207 Dini, U., 489 Domokos, M., 53 Domosi, P., 44 Drasny, G., 53 Druszt, E., 34
E Elbert, A., 50 Elek, G., 52 Engelking, R., 533, 537 Eotvos L., v Erdelyi, A., 183 Erdos, J., 11, 14, 19 Erdos, L., 52, 53 ror
INDEX OF NAMES
566
Erdos, P., 7, 11, 12, 15, 16, 19, 23, 26, 33, 36, 38, 43, 44, 145, 522, 558, 561
F Fazekas, I., 39, 45 Fejer, L., v, 151, 479 Fejes-Toth, G., 30, 40 Fejes-Toth, L., 4, 7, 9, 11, 13, 18, 20, 24, 27, 30, 36, 40 Feller, W., 406 Fischer, E., 471 Fleiner, T., 53 Foia§, C., 396 Frankl, P., 51 Freud, R., 50 Fried, E., 2, 7, 9, 17, 23, 27, 41 Fritz, J., 50 Fuchs, L., 2 Fi redi, Z., 51
Hajos, Gy., 3 Halasz, G., 18, 21, 24, 30, 33, 36, 40, 46, 50, 465, 467 Halmos, P. R., 348 Harcos, G., 53 Hardy, G. H., 217, 470, 484 Hatvani, L., 23, 29, 41, 48, 190 Hausel, T., 53 Hayman, W. K., 182 Heppes, A., 7 Hetyei, G., 52 Hewitt, E., 440 Horvath, L., 35 Hosszu, M., 3 Huhn, A., 31, 35 Huppert, B., 107
I Ivanyos, G., 52
J G
Gacs, P., 50, 51, 473, 477, 479 Galvin, F., 17, 30 Garay, B., 44 Geher, L., 9 Gelfand, I. M., 459 Gereb, M., 51, 52 Gerencser, L., 50 Gerlits, J., 553 Gesztelyi, E., 12, 26, 32, 37 Gondocs, F., 51 Gorenstein, D., 104 Greenleaf, F. R., 194 Gyires, B., 12, 20 Gyori, E., vii, 51 Gyori, I., 15 Gydry, K., 11, 25, 38
H Haar, A., v Hajdu, G., 53 Hajnal, A., 7, 10, 14, 20, 21, 23, 27, 32, 35, 39, 522, 558, 561
Hajnal, P., 52
Jaglom, I. M., 184 Joo, I., 36, 43 Juhasz, I., 9, 13, 24, 27, 31, 46, 50, 518, 520, 554
K Kalina, J., 27 Karman, T., v Karolyi, Gy., 52 Karteszi, F., 25 Katai, I., 10, 38, 513 Katona, Gy., 23, 39, 50 Keleti, T., 53 Kelly, D., 39
Kemperman, J. B. H., 336 Kennedy, P. B., 182 Kerchy, L., 29, 41, 48 Kertesz, A., 19, 84 Kery, G., 50 Kiss, E., 51, 52, 538 Knuth, E., 50 Koljada, K. I., 35 Kollar, J., 51, 56, 540 Komjath, P., vii, 42, 51, 52
INDEX OF NAMES
Komlos, J., 21, 50 Komornik, V., 51, 297 Kos, G., 53 Kovics, B., 38 Kovics, I., 9 Kovics, S., 52, 53 Krisztin, T., 34, 47, 48, 51, 52, 242
L
Laczkovich, M., 17, 24, 27, 33, 36, 39, 43, 44, 45, 50, 51 Langberg, N. A., 443 Laver, R., 559, 560 Leindler, L., 8, 15, 16, 22, 47, 491 Lempert, L., 24, 27, 43, 51 Leon, R. V., 443 Levi, B., 439 Linnik, Yu. V., 107 Littlewood, J. E., 470 Losonczi, L., 12, 26 Lovasz, L., 20, 22, 23, 28, 29, 36, 50, 51, 263, 415, 473, 477, 479, 524, 553
Lukics, E., 42
567
Montagh, B., 113 Moor, A., 9 Mori, T. F., 32, 33, 37, 43, 46, 51 Moricz, F., 16, 29, 47, 438 Moussong, G., vii, 52, 545
N Nagy, P., 29 Nagy, Zs., 51 Neumann, J., v Novikov, S. P., 82 Natanson, I. P., 168, 479 Nemetz, T., 465
0 Odor, T., 52
P Pach, J., 43 Pales, Zs., 31, 38, 52 Palfy, P. P., 31, 36, 39, 51, 52, 297 Pap, Gy., 37 Pelikan, J., vii, 17, 20, 30, 50, 51, 81
M Magyar, A., 52, 53 Magyar, Z., 51, 52 Major, P., 21, 450, 459 Majoros, L., 53 Makai, E., 18, 24, 50, 51, 477 Makay, G., 53 Makkai, M., 10, 13 Maksa, Gy., 25, 38 Marki, L., 28 Mate, A., 11, 15, 50, 491, 519, 521, 558, 561
Mate, E., 50 McLean, R. P., 85 Medgyessy, P., 19 Megyesi, L., 22 Michaletzki, Gy., 40 Miklos, D., vi, 52 Mocsy, M., 52, 53 Mogyorodi, J., 425
Petruska, Gy., 17, 50 Pinkus, A., 36, 209 Pinter, L., 29 Pintz, J., 51 Pollak, Gy., 3, 41 Polya, Gy., 180, 462, 470, 492 Pontriagin, L. S., 189 P6sa, L., 23, 28, 50, 51, 479, 520, 528, 553
Prachar, K., 107 Prekopa, A., 425 Preston, G. B., 85 Proschan, F., 443 Pyber, L., 42, 45
R Rado, R., 145, 522, 558, 561 Rado, T., v Ramsey, F. P., 122, 134
Ritz, L., v Redei, L., 3, 8, 13, 25, 103
INDEX OF NAMES
568
Renyi, A., 3, 4, 8, 11, 425, 433, 443, 453, 461 Revesz, P., 14, 19, 24, 37, 443 Revesz, Sz. Gy., 41 Reviczky, J., 51 Riesz, F., 394 Riesz, M., v
Riman, J., 38 Rudin, M. E., 545 Rudin, W., 505 Ruzsa, I. Z., vii, 27, 39, 45, 50, 51, 440, 445, 477 S
Saks, S., 330, 336 Salem, R., 479 Schrijver, A. A., 28 Schottky, W., 228 Schur, I., 382 Schweitzer, M., v, vi Seb6, A., 51 Seress, A., 52 Shapiro, H. S., 27 Shilow, G. E., 459 Shockey, H., 207 Sidon, S., 135 Sigray, I., 52, 53 Simanyi, N., 51, 52, 290, 538 Simonovits, A., 477, 484 Simonovits, M., 50, 470, 519, 553, 554
Somorjai, G., 27, 51, 501 Sos, V. T., 30, 33, 34 Stromberg, K., 440 Suranyi, J., 2, 9, 13, 18 Sylvester, J. J., 382 Szabados, J., 22, 24, 30, 40, 45 Szabo, E., 52 Szabo, Gy., 38, 44 Szab6, L. I., 46, 52 Szabo, T., 53, 513 Szab6, Z., 28, 34, 35, 52, 53, 539 Szasz, D., 50, 425 Szegedy, M., 52 Szeg6, G., v, 180, 492 Szekely, G. J., vi, 32, 440, 445
Szekely, L., 34, 36 Szekelyhidi, L., 25, 44 Szekeres, Gy., 7, 30, 33, 40 Szemeredi, E., 27 Szendrei, A., 41, 47 Szendrei, M., 51 Szenes, A., 52 Szenthe, J., 16 Szep, G., vii Szigeti, F., 50 Szilard, L., v Szilasi, J., 39, 45 Szokefalvi-Nagy, B., 4, 9, 22, 394, 396
Szucs, J., 22, 23, 31, 35, 50
T Tamassy, L., 12, 25 Tandori, K., 4 Tardos, G., 52 Teller, E., v Terjeki, J., 34 Tomko, J., 26 Torocsik, J., vii, 52, 53 Totik, V., vii, 29, 34, 36, 39, 41, 42, 46, 47, 48, 51, 209, 297, 539
Turin, P., 2, 3, 8, 10, 14, 18, 158, 431
Tusnady, G., 14 Tuza, Zs., 51, 297
V Vamos, P., 50 Varga, J., 52 Varlet, J., 27 Veres, S., 51 Vesztergombi, Gy., 50, 519 Vitali, G., 336, 510 Vu, H. V., 53
W Weiss, B., 39 Wiegandt, R., 34, 43 Wielandt, H., 96, 97
Wigner, E., v
Zempleni, A., 52 Zenor, P. H., 545 Zorn, M., 86, 345
Problem Books in Mathematics
(continued)
Demography Through Problems by Nathan Keyfitz and John A. Beekman
Theorems and Problems in Functional Analysis by A.A. Kirillov and A.D. Gvishiani
Exercises in Classical Ring Theory by T. Y. Lam
Problem-Solving Through Problems by Loren C. Larson
A Problem Seminar by Donald J. Newman
Exercises in Number Theory by D.P. Parent
Contests in Higher Mathematics: Miklos Schweitzer Competitions 1962-1991 by Gabor J. Szekely (editor)